数学分析 I 速通日志(下)

隔了几天才发是因为想不出来摘要可以写些什么,现在也还没想出来,所以就这样吧。差不多得了(

最近看到多元统计分析的先修要求包含“熟练运用 R”,给我整麻了,去推了几天 R 的进度,把数分耽搁了。我好菜。

如果还要多写几句的话,讲点近期见闻、甘主任的小故事以及毫无意义的竞赛往事!(

前两天看到 Chenlin Gu 的 homepage 上有这么一句话:

Finally, let mention the guy who teaches me a lot. We have once trained for maths olympiad in high school and also collaborated for (maybe) my first paper. I always remember the epoch when I was younger and those friends with who we studied maths together. I missed the old days, but perhaps to do maths is the only way to keep them in my mind.

——Chenlin Gu's homepage

然后我想了想,若干年后我会不会也可以原文照抄这段话放在自己的关于页面里,然后链接到甘主任的 homepage(。前三句现在基本已经实现了(如果把我们高中的时候在新星那里生产的学术垃圾题解当做 maybe my first paper 的话,那就真的已经完全实现了)。听他之后的想法的话,最后一句或许也是有可能的。当然大家的想法都会慢慢变化,说不定将来在自己的 homepage 上写这句话然后链接到我的主页的人是他呢(笑

后来觉得很奇怪。我在进高中之前机缘巧合认识了甘主任,又过了两年直到退役也没在学校里找到新的学习数学的朋友。尽管我校也不算什么强校,但数学组加起来也有几十个人,这里面的人除了一些耳熟能详的神和同班同学之外,我几乎都不认识,还是只能和甘主任讨论问题。

不过有一个好朋友也该知足了。高一联赛时刚学了几个月,我属于是超常发挥,和彼时初三的甘主任拿到了相同的分数,大概是省二靠前的位置,一度因此产生“再学两年我一定可以和甘主任同时进省队”的幻觉(我始终认为自己的上限就是省队,所谓求上者得中,求中者得下,我就是那个求下者不入流的),甚至当时被教练当成种子选手之一,现在想想很对不起唐老师。第二年甘主任如愿拿到了省一,而我因为各种原因(精神状态很差,具体情况在此略过)耻辱省三。后来我的水平就被甘主任拉得越来越远,半退役了很长时间,还碰上了疫情和网课,变成了坚持到底就是胜利的乐子人。我高三的时候甘主任大约只差了五个名次没有进队,而我拿了个牌拿了个垫底省一就退役了,彻底离开了这个我当时已经并不特别喜欢的游戏。不喜欢的原因可以有很多,两个最大的因素是觉得还是在练习做题技巧,有点无聊,以及自己相对别人的基础实在差距太大,也不太聪明,感觉付出更多努力也弥补不了多少。高三的时候有一次我做梦梦到班主任问我为什么高二都摆过去了没有好好搞竞赛,我说我不喜欢这个游戏规则了,然后他对我嗤之以鼻,说你是学生,不喜欢也得搞。醒来之后我觉得这对话还挺符合他形象的,男人至死是少年,所以他当了老师也还是不停强调自己当年是理科状元考进了清华所以会做题最牛逼也没什么不对的。当然现实里的话他根本不会和我多啰嗦一个字,几乎没有和我说过话。

扯远了。拍毕业照的时候我把 CGMO 的金牌挂在甘主任脖子上和他合了影,半年后就把他奶成了货真价实的 CMO Au,顺利签了隔壁数院的英才班。我常开玩笑说是我的破铁牌的功劳,建议请我吃饭,心里时不时想我清醒地知道自己怎么样也不可能在这场游戏里和他达到同样的水平,愿赌服输。无论是资源、基础这种已经改变不了了的东西,还是实在的努力,甚至耐心、心态、韧性这些听起来很玄学的因素,或许还有听起来很扎心的智力,我承认自己哪一点都比不过他。甚至我觉得,我的心早在退役之前就已经飞走了,在最后两次比赛之前就已经不怎么喜欢这个游戏了,只是在为沉没成本找补,以及试图最后寻找到一点独立解题的乐趣。当然寻找乐趣这件事还是令人欣慰的,最后半年的快乐回忆包括并不限于质心首届线上联考手推 Chebyshev 多项式拿了全国 rank 1(主要是全国也没几个人参加这个破玩意),CGMO Day 1 迅速切掉了压轴题,联赛那天只睡了不到五个小时还是几乎 ak 了一试,都是前所未有的愉快体验。当然这不能说明我水平怎么样,熟悉我的人都知道我无论是从前还是现在都很菜,大大小小的比赛和日常练习里失败的次数远远多于成功的,敝帚自珍而已。

又扯远了,我在说甘主任来着。总之,他是我高中的时候关系最好的同路的朋友,没有之一,现在或许也是。尽管我高中竞赛是真的学得很艰难,他也没嫌弃过我菜还随时解答我的奇怪问题,泪目了,也收到过不少他的鼓励(或者说毒奶),虽然毒奶里面只实现了签清华的约这件事情,强基之后还自动毁约了。运气不好被华子调剂了专业之后,他和我说过“有些人偶尔走失,但从未离开”,我当时一心转码,正处在一个走失的状态,没有意识到这句话的可贵之处。

可惜甘主任目前还没有自建的主页或者博客,先浅放一个他的公众号在这里:runzhi的随笔(我不太会直接链接公众号的界面,就放了一篇最近的文章的链接),感觉功能上也是一样的。一个多月没更新了,隔空催更一下(不

最后贴一个今晚躺着刷知乎看到的回答,笑得想死,希望我没事:

no-analysis.png

导数

2023 年 2 月 1 日,星期三,学习一些轻松愉快的导数。

我先简单喵两句

  • 一个处处不可微的连续函数的例子,不出意外地是无穷级数函数,通俗一点来说处处不可微是因为求导之后的级数就不收敛了。但是对无穷级数的求导暂时不好定义,所以是取一个收敛的点列来论证差分的极限发散。直接贴这儿吧:Read More

  • 另外观察导数定义的形式:\(\lim _{x \to x_0} \frac{f(x)-f(x_0)}{x-x_0}\),我们可以知道只要 \(f: \mathbb R \to V\) 是连续映射,\((V,|| \cdot ||)\) 是赋范线性空间,那么这个极限就有意义,可以定义导数。比如说映射 \(x \to exp (xA), \mathbb R \to M_n(\mathbb R)\) 就有导数。

  • 一些可能没学过或者单纯是忘了的定理:

    导函数不一定是连续的,但是它可以满足介值定理。所以 Cauchy 中值定理的形式 \(\frac{f'(x)}{g'(x)}\) 实际上意味着 \(g(x)\) 有比 \(f(x)\) 更强的性质,\(g'(x) \neq 0\),也就是说它是恒正或者恒负的,是单调且连续的,也就是一个连续的双射而且满足逆映射也连续,所以是一个同胚。

    Cauchy 中值公式可以看做 Lagrange 中值定理在二维上的推广:对于映射 \(x \to (f(x),g(x))\)\(I\) 上两点 \(a,b\),总可以找到一点 \(x_0 \in [a,b]\) 满足 \((f(a),g(a))\)\((f(b),g(b))\) 的斜率等于 \(x_0\) 处的切线斜率,也就是 \((f'(x_0),g'(x_0))\)

  • 然后是一些凸函数理论,挺应用的,有手就行,不细看了。

其他没啥可写的,跑步进入习题环节!

梦话:经典的作业题胡说八道环节

导数的定义和计算

  • \(\mathbb R^n\) 上配有范数 \(\|(x_1,\cdots,x_n)\|_2=\sqrt{(x_1)^2+\cdots+(x_n)^2}\),我们考虑映射

    \(f\colon \mathbb R \rightarrow \mathbb R^n, \ \ x\mapsto f(x)=(f_1(x),\cdots, f_n(x))\)

    证明,\(f\)\(x_0\) 处的导数存在当且仅当对每个分量函数 \(f_k\),它在 \(x_0\) 处的导数都存在并且

    \(f'(x)=\bigl(f_1'(x),\cdots, f_n'(x)\bigr)\)

    Trivial.

  • 考虑函数 \(e^{ix}\),将它视作是映射 \(f\colon \mathbb R\rightarrow \mathbb{C}, \ \ x \mapsto e^{ix}\)

    利用定义证明,\(f'(0)=i ~(\in \mathbb{C})\)。仿照课堂上的做法证明 \(\bigl(e^{ix}\bigr)'=ie^{ix}\)

    老演员了,PIN 真的很喜欢 exp 函数。

  • 利用上面两个问题的结论计算 \(\sin x\)\(\cos x\) 的导数。

    \(f:\mathbb R \to \mathbb R^2\)\(x \to (sinx,cosx)\),(?),把二者都写成 \(e^{ix}\) 的变式。

  • 定义映射 \(E\colon \mathbb R \rightarrow \mathbb{C}=\mathbb R^2, \ \ \theta\mapsto (\cos\theta, \sin\theta)\),我们用下面符号表示平面上的单位圆:

    \(\mathbf{S}^1=\bigl\{(x,y)\in \mathbb R^2\bigm| x^2+y^2=1\bigr\}\)

    证明,\(\mathbf{S}^1\) 上的点可以写成 \((\sin\theta,\cos\theta)\) 的形式,即 \(E(\mathbb R)=\mathbf{S}^1\)。试计算 \(E'(\theta)\) 并验证对于这个映射(\(\mathbb R^2\) 上取值),Rolle中值定理并不成立。

    圆上的点可以视作一条过原点的直线和圆的交点,所以问题的重点是为什么 \(\frac{\cos \theta}{\sin \theta}\) 可以表示一切实数,换成更熟悉的形式就是为什么 \(\tan \theta\) 可以表示一切实数。实际上 \(\tan x = x + \frac{x^3}{3!}+...\) 是发散的无穷级数,而且是连续函数(因为 \(\sin x\)\(\cos x\) 都是连续函数,导出 \(\tan x\) 也是连续函数),所以它可以表示一切实数。

    \(E'(\theta) = (\cos \theta , -\sin \theta)\),考虑 Rolle 中值定理,有 \(E(0) = E(2 \pi)\),但是不存在 \(x \in [0,2\pi]\) 满足 \(E'(x) = (0,0)\)。所以 Rolle 中值定理是只能对一维情况成立的。

  • 如果函数 \(f\) 在点 \(x_0\) 处的导数 \(>0\),不能推出存在该点的领域 \(U\),使得 \(f\) 在这个邻域上是递增的:

    考虑函数\(f(x)=\begin{cases} x+2x^2\sin\left(\frac{1}{x}\right), \ \ &x\neq 0;\\ 0, \ \ &x=0. \end{cases}\)

    证明,\(f\)\(0\) 处导数存在且大于零,但是对任意的 \(\varepsilon>0\)\(f\)\((-\varepsilon,\varepsilon)\) 上的限制都不是单调函数。

    \(f\)\(0\) 处的导数值是 \(1\)。第二问要证明的意思其实是,虽然 \(f'(0) >0\) 但是在任意的邻域上不单调。

    \(x \neq 0\) 时的导函数是 \(f'(x) = 1+4x \sin (\frac{1}{x}) -2 \cos (\frac 1 x)\),在任意的邻域 \((-\varepsilon , \varepsilon)\) 上不保证大于 \(0\)

  • \(\mathbf{I}_n\)\(n\times n\) 的单位矩阵,\(A\in \mathbf{M}_n(\mathbb R)\),计算 \(\dfrac{d}{dx}\Big|_{x=0}\det (\mathbf{I}+xA)\),即 \(\det (\mathbf{I}+xA)\)\(x=0\) 处的导数。

    从特征值的角度考虑就很 trivial 了,是 \(trace (A)\)

  • 证明,(可微的)奇函数的导数是偶函数,(可微的)偶函数的导数是奇函数。

    Trivial.

  • 证明,Riemann 函数 \(f(x) = \begin{cases} 1/q, &x = \dfrac{p}{q} \in \mathbb{Q}, q\geqslant 1, p~\text{和}~q~\text{互素};\\0, & x~\text{是无理数}.\end{cases}\)\(\mathbb{R}\) 上处处不可微.

    在有理数点处不连续自然不可微。无理数点处可以找一个收敛到 \(x\) 的有理序列,逐点相对于 \(x\) 的差商可以递增,不会收敛,所以也不可微。

    具体一点来说,对于任意的素数 \(q_n\),取长度为 \(\frac 1 {q_n}\) 的区间使得 \(x\) 落在其中,可以表示成 \([a_n,b_n]=[\frac{[qx]}{q},\frac{[qx]+1}{q}]\),我们总是取 \(a_n,b_n\) 中距离 \(x\) 更近的一点作为 \(x_n\),由有理数的稠密性可以知道,$ = $ 可以取成递增发散的序列。

无穷小量与无穷大量的阶的比较

如果函数 \(f\)\(x_0\) 的附近(即某个 \(x_0\) 的开邻域去掉 \(x_0\))满足 \(\displaystyle\lim_{x \to x_0} f(x) = 0\),我们就称 \(f\)\(x \to x_0\) 时的无穷小量; 类似地,如果 \(\displaystyle\lim_{x \to x_0} f(x) = +\infty\) 或者 \(\displaystyle\lim_{x \to x_0} f(x) = -\infty\)(其注意我们用的“或者”这个词的含义),我们就称 \(f\)\(x \to x_0\) 时的无穷大量。

现在假设 \(f, g\) 都是 \(x \to x_0\) 时的无穷小量并且 \(g(x)\)\(x_0\) 的附近不取零值,我们现在引进记号:

  • 如果 \(\displaystyle\lim_{x \to x_0}\frac{f(x)}{g(x)} = 0\), 我们就称 \(f\) 是比 \(g\) 高阶的无穷小,记作 \(f(x) = o(g(x)), x \to x_0\)

  • 如果 \(\displaystyle\lim_{x \to x_0}\frac{f(x)}{g(x)} = \ell, \ell \neq 0\),我们就称 \(f\) 是与 \(g\) 同阶的无穷小;

  • 特别地,如果 \(f\)\(g\) 同阶并且 \(\ell = 1\),我们就称 \(f\) 是与 \(g\) 等价的无穷小, 记作 \(f(x) \sim g(x), x \to x_0\)

  • 如果 \(\displaystyle\limsup_{x \to x_0}\frac{|f(x)|}{|g(x)|} < +\infty\),我们将这种情况记作 \(f(x) = O(g(x)), x \to x_0\)

特别的 \(f(x) = O(1)\)\(f(x) = o(1)\) 分别表示在 \(x \to x_0\)\(f\) 有界与 \(f\) 是无穷小.

类似地,我们可以定义无穷大量的阶之间的比较。这是通用的术语,同学们可参考任一本参考书或者网络。

  • 假设 \(x \to x_0\) 时,函数 \(a(x)\) 满足 \(a = o(1)\)。试证明:

    \(o(a) + o(a) = o(a)\)

    $ o(ca) = co(a), c $

    \((o(a)^k) = o(a^k)\)

    $ = 1 - a + o(a)$

    代定义就可以了,trivial.

  • 假设 \(f(x), g(x)\)\(x \to x_0\) 时的无穷小,那么

    • 证明,\(f(x) \sim g(x) \iff f(x) - g(x) = o(g(x)), \quad x \to x_0\)

      Trivial.

    • 如果把 \(g\) 作为基本的比较单位(小量),我们可以将另外的无穷小量与 \(g(x)^k\)\(k\) 是正整数)进行比较。如果 \(f(x) \sim c g(x)^k\),我们就称 \(c g^k\)\(f\) 的主部(这里 \(c\) 是常数)。试定出下列无穷小或无穷大的主部(与 \(x-x_0\) 或者 \(x\) 比较):

      \(\frac{1}{\sin \pi x}, \quad x \to 1\)

      这是个无穷大量,或许主部是 \(\frac 1 \pi (1-x)^{-1}\)

      \(\sqrt{1+x} - \sqrt{1-x}, \quad x \to 0\)

      主部是 \(x\)

      \(\sin \left(\sqrt{1 + \sqrt{1+ \sqrt{x}}} - \sqrt{2}\right), \quad x \to 0^+\)

      主部是 \(\frac{1}{4 \sqrt{2} } \sqrt{x}\),所以说 \(k\) 真的一定是正整数吗?

      \(\sqrt{1 + \tan x} - \sqrt{1 - \sin x}, \quad x\to 0\)

      主部是 \(x\)

      \(\sqrt{x + \sqrt{x + \sqrt{x}}}, \quad x \to 0^+\)

      主部是啥?

      \(\sqrt{x + \sqrt{x + \sqrt{x}}}, \quad x \to +\infty\)

      主部是 \(\sqrt{x}\)

  • 我们假设 \(f(x) \sim c x^k, x \to 0\)(即 \(f(x) = c x^k + o(x^k)\))。如果 \(f(x) - c x^k\) 可以再分出主部 \(c'x^{k'}\),其中 \(k' > k\),那么我们就将它写为 $f(x) = c x^k + c' x^{k'} + o(x^{k'}) $。试将下列无穷小展开到 \(o(x^2)\)

    • \(\sqrt{1 + x} - 1\)

      \(\frac 1 2 x - \frac 1 4 x^2\)

    • \(\sqrt[m]{1+x} - 1, m \in \mathbb{Z}_{\geqslant 1}\)

      \(\frac 1 m x + \frac{1-m}{m^2} x\)

*高木贞治函数(学不完了,咕了

我们先在 \([0,1]\) 区间上定义 \(\psi(x)=\begin{cases}x, \ \ \ \ 0\leqslant x <\dfrac{1}{2};\\1-x, \ \ \dfrac{1}{2}\leqslant x \leqslant 1.\end{cases}\)

接下来,以 \(1\) 为周期,我们可以将 \(\psi\) 延拓成 \(\mathbb R\) 上的周期函数(连续)并且仍然将它记作 \(\psi\),它的函数图像好像是锯齿一般。

我们定义Takagi函数 \(T\colon \mathbb R\rightarrow \mathbb R\) 如下:\(T(x)=\sum_{k=0}^\infty \frac{1}{2^k}\psi(2^k x)\)

我们实际上可以考虑部分和 \(\displaystyle T_n(x)=\sum_{k=0}^n \frac{1}{2^k}\psi(2^k x)\)。当 \(n\) 越来越大的时候,它们的图像看起来逐渐地收敛。这个习题的目标是粗略地研究Takagi函数的性质。

  • 证明,\(T(x)\)\(\mathbb R\) 上良好定义的有界连续函数。

    良好定义:\(T(x) \leq \sum_{k=0}^\infty \frac{1}{2^k} = 2\),一定是收敛级数,所以是良好定义的。

    有界:如上所述。

    连续函数:它是连续函数的级数和,也是连续函数。

  • 对于 \(x\in [0,1]\),假设 \(x=\displaystyle\sum_{n=1}^\infty \frac{a_n}{2^n}\)\(x\)\(2\)-进制展开,其中 \(a_n=0\) 或者 \(1\)。我们令 \(v_n=\displaystyle\sum_{k=1}^n a_k\)。我们定义函数 \(\sigma_n(y)=a_n+(1-2a_n)y\),其中 \(y=0\) 或者 \(1\)。证明,\(\psi(2^m x)=\sum_{n=1}^\infty\frac{\sigma_{m+1}(a_{m+n})}{2^n}\)

    Trivial.

  • 对于 \(x\in [0,1]\),假设 \(x=\displaystyle\sum_{n=1}^\infty \frac{a_n}{2^n}\)\(x\)\(2\)-进制展开。证明,\(T(x)=\sum_{n=1}^\infty\frac{(1-a_n)v_n+a_n(n-v_n)}{2^n}\)

  • 假设 \(x_0=\dfrac{k_0}{2^{m_0}}\in(0.1]\),其中 \(k_0\in\mathbb{Z}_{\geqslant 1}\) 是奇数,\(m_0\in \mathbb{Z}_{\geqslant 0}\)。令 \(h_n=\dfrac{1}{2^n}\),其中 \(n\in \mathbb{Z}_{\geqslant m_0}\)。证明,数列 \(\bigl\{\dfrac{T(x+h_n)-T(x)}{h_n}\bigr\}_{n\geqslant m_0}\) 不收敛。

  • \(f\) 是定义在非空的开区间 \(I\) 上实数值函数。如果 \(f\)\(a\) 处可导,

    证明,\(\lim_{(h,h')\rightarrow (0,0), \atop \\ h>0,h'>0}\frac{f(a+h)-f(a-h')}{h+h'}=f'(a)\)

    这里极限 \(\displaystyle \lim_{(h,h')\rightarrow (0,0), \atop \\ h>0,h'>0}\) 的意义指的是任意的序列 \((h_n,h'_n)\rightarrow (0,0), h_n>0,h'_n>0\) 所对应的序列都收敛。

  • \(f\) 是定义在非空的开区间 \(I\) 上实数值函数。假设 \(f\in C^1(I)\)(连续可微),\(a\in I\)

证明,\(\lim_{(h,h')\rightarrow (0,0), \atop h+h'\neq 0}\frac{f(a+h)-f(a-h')}{h+h'}=f'(a)\)

  • 假设 \(x\in [0,1]\),使得对任意的正整数 \(n\)\(2^n x\) 都不是整数。对于每个正整数 \(n\),我们用下面的公式定义序列 \(\{h_n\}_{n \geqslant 1}\)\(\{h'_n\}_{n \geqslant 1}\)\(\floor{2^n x}=2^n(x-h'_n), \ \ \floor{2^n x}+1=2^n(x+h_n)\)

    其中函数 \(\floor{y}\) 按照定义是不超过 \(y\) 的最大的整数(即 \(y\) 的整数部分(如果 \(y\geqslant 0\)))。证明,对每个给定的 \(n\)\(h_n+h'_n=2^{-n}\) 并且对每个整数 \(1\leqslant \ell \leqslant n-1\),开区间 \(\bigl(2^\ell(x-h'_n),2^\ell(x+h_n)\bigr)\) 中不包含任何的整数和半整数。

  • 假设 \(x\in [0,1]\),使得对任意的正整数 \(n\)\(2^n x\) 都不是整数,我们沿用~E7)中的符号,

    证明,数列 \(\Bigl\{\dfrac{T(x+h_n)-T(x-h'_n)}{h_n+h'_n}\Bigr\}_{n\geqslant 1}\) 不收敛。

  • 证明,\(T(x)\)\(\mathbb R\) 上处处连续处处不可微的函数。

  • 证明,我们有如下的函数方程:

    \(T(x)=\begin{cases}2x+\dfrac{T(4x)}{4},\ \ 0\leqslant x <\dfrac{1}{4};\\\dfrac{1}{2}+\dfrac{T(4x-1)}{4},\ \ \dfrac{1}{4}\leqslant x <\dfrac{1}{2};\\\dfrac{1}{2}+\dfrac{T(4x-2)}{4},\ \ \dfrac{1}{2}\leqslant x <\dfrac{3}{4};\\2-2x+\dfrac{T(4x-3)}{4},\ \ \dfrac{3}{4}\leqslant x \leqslant 1.\end{cases}\)

  • (Takagi函数图像的自相似性)令 \(\Gamma=\bigl\{(x,T(x))\mid 0\leqslant x\leqslant 1\bigr\}\subset \mathbb R^2\)\(T\) 在区间 \([0,1]\) 上的函数图像。我们定义如下四个仿射变换 \(\Phi_i\colon \mathbb R^2\rightarrow \mathbb R^2\)

    • \(\Phi_0 \left(\begin{array}{ccc}x \\y \end{array} \right)&= \left(\begin{array}{ccc}\frac{1}{4} & 0 \\\frac{1}{2} & \frac{1}{4} \end{array} \right)\left(\begin{array}{ccc}x \\y \end{array} \right)\)

    • \(\Phi_1 \left(\begin{array}{ccc}x \\y \end{array} \right)= \left(\begin{array}{ccc}\frac{1}{4} & 0 \\0 & \frac{1}{4} \end{array} \right)\left(\begin{array}{ccc}x \\y \end{array} \right)+\left(\begin{array}{ccc}\frac{1}{4} \\\frac{1}{2}\end{array}\right)\)

    • \(\Phi_2 \left(\begin{array}{ccc}x \\y \end{array} \right)&= \left(\begin{array}{ccc}\frac{1}{4} & 0 \\0 & \frac{1}{4} \end{array} \right)\left(\begin{array}{ccc}x \\y \end{array} \right)+\left(\begin{array}{ccc}\frac{1}{2} \\\frac{1}{2}\end{array}\right)\)

    • \(\Phi_3 \left(\begin{array}{ccc}x \\y \end{array} \right)= \left(\begin{array}{ccc}\frac{1}{4} & 0 \\-\frac{1}{2} & \frac{1}{4} \end{array} \right)\left(\begin{array}{ccc}x \\y \end{array} \right)+\left(\begin{array}{ccc}\frac{3}{4} \\\frac{1}{2}\end{array}\right)\)

      证明,\(\Phi_i\) 恰好把 \(\Gamma\) 变成 \(T\) 在区间 \([\dfrac{i}{4},\dfrac{i+1}{4}]\) 上的图像,其中 \(i=0,1,2,3\)

  • \(S_0=\{(x,y)\in \mathbb R^2 \mid 0\leqslant x\leqslant 1, 0\leqslant y \leqslant 1\}\)\(\mathbb R^2\) 上的闭方块。对每个 \(n\geqslant 0\),我们定义 \(\displaystyle S_{n+1}=\bigcup_{k=0}^3\Phi_k(S_n)\)。证明,\(S_n\) 是平面上一列单调下降的紧集并且 \(\displaystyle \Gamma=\bigcap_{n\geqslant 0}S_n\)。我们有 \(S_1\)\(S_2\) 的示意图:

  • 证明,\(\displaystyle\sup_{x\in \mathbb R}T(x) \leqslant \dfrac{2}{3}\)

  • 找一个 \(c\in [0,1]\),使得 \(T(c)=\dfrac{2}{3}\)

  • \(T^{-1}(\dfrac{2}{3})\) 的Cantor集的结构)对于 \(x\in [0,1]\),假设 \(x=\displaystyle\sum_{n=1}^\infty \frac{b_n}{4^n}\)\(x\)\(4\)-进制展开,其中 \(b_n=0,1,2,3\)

    证明,\(\Bigl\{x\in [0,1]\bigm| T(x)=\dfrac{2}{3}\Bigr\}=\Bigl\{x\in [0,1]\bigm| x=\sum_{n=1}^\infty \frac{b_n}{4^n}, b_n\in \{1,2\}\Bigr\}\)

  • 仿照 11,研究 \(\Phi_1\)\(\Phi_2\) 在集合 \(\bigl\{(x,T(x))\bigm| x\in [0,1],T(x)=\dfrac{2}{3}\bigr\}\) 上自相似的作用。(这是一个Hausdorff维数为 \(\dfrac{1}{2}\) 的集合)。

梦话 2.0:这作业怎么这么多啊

中值定理和Taylor展开

如果不额外说明,\(f\) 总代表一个区间 \(I\) 上定义的函数。

  • \(f\) 在点 \(x\) 处有二阶导数。证明,下面的极限给出了二阶导数:$ f''(x) = _{h } $

    可以用定义,也可以直接洛。

  • (Taylor 展开式的唯一性,Peano余项)。假设 \(f\)\(x_0\) 附近的函数,并且当 \(x\rightarrow x_0\) 是,满足

    $ f(x) &= a_0 + a_1 (x-x_0) + a_2 (x-x_0)^2 + + a_n (x-x_0)^n + o(|x-x_0|^n)\ &= b_0 + b_1 (x-x_0) + b_2 (x-x_0)^2 + + b_n (x-x_0)^n + o(|x-x_0|^n)$

    其中 \(a_i, b_i, i=0,\cdots,n\) 是实数,那么,对任意的 \(i\),我们都有 \(a_i=b_i\)

    \(\lim _{x \to x_0} \frac{A(x)-B(x)}{(x-x_0)^n} =0\),于是有 \(a_0 = b_0,...,a_{n-1}= b_{n-1}\),最后可以推出 \(a_n = b_n\)

  • 假设 \(f\)\(0\) 处有 \(n\) 阶导数。证明,如果 \(f(x)\) 是偶函数(奇函数), \(f\)\(0\) 处的Taylor展开式(Peano余项)中只有 \(x\) 的偶次项(奇数项)。

    Trivial.

  • (Rolle定理的简单推广)设函数在有限或无穷的区间 \((a,b)\) 上可微并且 \(\displaystyle\lim_{x \to a^+} f(x) = \lim_{x \to b^-} f(x)\)。证明,存在 \(x_0 \in (a,b)\),使得 \(f'(x_0) = 0\)

    如果不存在,那么 \(f'(x)\)\((a,b)\) 上恒正或恒负,\(f(x)\)\((a,b)\) 上是严格单调函数,矛盾。

  • 设函数 \(f \in C^0([a,b])\) 并且在 \((a,b)\) 上可微。证明,\(f\)\([a,b]\) 上严格递增的充分必要条件是对任意 \(x\in (a,b)\)\(f'(x) \geqslant 0\) 并且在任意子区间 \((c,d) \subset (a,b)\) 上,\(f'(x)\) 不恒等于 \(0\)

    如果 \(f(x)\)\([a,b]\) 上严格单调递增,那么这两个结论很好推。

    如果满足这两个条件,那么至少有 \(f(x)\)\([a,b]\) 上非严格单调递增,可能存在 \(f(x_1) =f(x_2)\) 的情况,\(x_1 \neq x_2\)。此时对任意 \(s \in (x_1,x_2)\) 都有 \(f(s)=f(x_1)\),于是在这一子区间上 \(f'(x) =0\),矛盾。

  • \(g\in C(\mathbb R)\)\(\mathbb{R}\) 上可微。假设存在常数 \(M\),使得 \(\displaystyle\sup_{x \in \mathbb{R}}|g'(x)| \leqslant M\)。 对任意的 \(\varepsilon > 0\),我们定义 \(f_\varepsilon(x) = x + \varepsilon g(x)\),证明,存在仅依赖于 \(M\) 的常数 \(\delta = \delta(M) > 0\),使得当 \(\varepsilon < \delta\) 时,\(f\colon \mathbb{R} \rightarrow \mathbb{R}\) 是双射。

    \(\delta = \frac{1}{2M}\) 即可。

  • 设函数 \(f\)\([a,b]\) 上有两阶导数并且 \(f'(a) = f'(b) = 0\)。证明,存在 \(c \in (a,b)\) 使得 \(|f''(c)| \geqslant \frac{4}{(b-a)^2}|f(b) - f(a)|\)

    存在 \(s \in (a,b)\) 使得 \(f'(s) = \frac{f(b)-f(a)}{b-a}\),存在 \(t \in (a,s)\) 满足 \(f''(t) = \frac{f'(s)}{s-a} = \frac{f(b)-f(a)}{(b-a)(s-a)}\)

    于是 \(t\) 就是满足题意的 \(c\)

  • 假设 \(f\)\(\mathbb{R}\) 上二次可导,对 \(k=0,1,2\),我们假设 \(M_k = \displaystyle\sup_{x \in \mathbb{R}} |f^{(k)}(x)|\) 都是有限的。 证明,\(M_1^2 \leq 2 M_0 M_2\)

    随便写个 Lagrange 余项的泰勒展开:$f(x+h)= f(x)+f'(x)h+ 2 f''(t)h^2 \(,\)t (x,x+h)$。

    然后推一个不等式就行了。

  • 假设 \(f\)\((0,+\infty)\) 上二次可导,\(f''\)\((0,+\infty)\) 上有界并且 \(\displaystyle\lim_{x \rightarrow +\infty} f(x) = 0\)。证明,\(\displaystyle\lim_{x \to +\infty} f'(x) = 0\)

    随便写个 Lagrange 余项的泰勒展开:$f(x+h)= f(x)+f'(x)h+ 2 f''(t)h^2 \(,\)t (x,x+h)$。

    \(x \to + \infty\),于是此时 \(f'(x) =- \frac 1 2 f''(t) h +\frac{f(x+h)-f(x)}{h}\)。对任意的 \(\varepsilon >0\),只要取 \(|f(x+h)-f(x)|<\varepsilon ^2\)(Cauchy 收敛准则),取 \(h < \varepsilon\),就有 \(|f'(x)|<\frac{M\varepsilon}{2}\),说明它也会收敛到 \(0\)

极大值的判定

\(f\)\((a,b)\) 上可微。假设对于 \(x_0 \in (a,b)\),我们有 \(f'(x_0) = 0\)

  • 证明,\(f\)\(x_0\) 处取局部极大值的一个充分条件是:存在某一邻域 \((x_0 - \delta, x_0 + \delta) \subset (a,b)\),使得

    \(f'(x)= \begin{cases}< 0, \ \text{对所有的}~x \in (x_0-\delta, x_0);\\\\> 0, \ \text{对所有的}~ x \in (x_0, x_0+\delta).\end{cases}\)

    Trivial.

  • (最重要的判别方法,有很多应用)证明,如果 \(f''(x_0)\) 存在并且 \(f''(x_0) < 0\),那么 \(f\)\(x_0\) 处取局部极大值。

    说明在某个邻域 \((x_0 - \delta, x_0 + \delta) \subset (a,b)\) 有 $f''(x) $,也就是 \(f'(x)\) 在邻域上递减,也就是上题的情况。

  • 假定 \(f\)\(x_0\) 处有 \(n\) 阶导数,\(f'(x_0) = \cdots = f^{(n-1)}(x_0) = 0\) 并且 \(f^{(n)}(x_0) \neq 0\),试讨论 \(f\)\(x_0\) 处取局部极大值的条件(对 \(n\) 分奇偶讨论)。

    \(f(x)-f(x_0) = \frac{1}{n!} f^{(n)} (x_0) (x-x_0)^n + o(|x-x_0|^n)\),所以 \(n\) 是奇数则不是局部最值,\(n\) 是偶数且 \(f^{(n)}(x_0) < 0\) 局部极大,否则局部极小。

多项式的根

(提示:利用中值定理和 \(n\)-次多项式的至多(恰好)有 \(n\) 个根)

  • 证明,如果实系数多项式 \(P_n(x) = a_n x^n + a_{n-1} x^{n-1} + \cdots + a_0\) 的根都是实数(不妨设 \(a_n\neq 0\)),那么它的逐次导函数 \(P_n'(x)\)\(P_n''(x)\)\(\cdots\)\(P_n^{(n-1)}(x)\) 的根也都是实数。

    只考虑一阶导数的根。\(P_n(x)\)\(n\) 个实数根(计入重根),Rolle 定理保证 \(P_n ' (x)\)\(n-1\) 个实数根(也可能计入了重根,这个并不重要),归纳即证。

  • 证明,Legendre 多项式 \(P_n(x) = \dfrac{1}{2^n n!} \dfrac{d^n}{dx^n}(x^2 - 1)^n\) 的根都是实数并且包含于区间 \((-1,1)\) 中。

    \(P(x) = (x^2-1)^n\) 的根都是实数,这是一个 \(2n\) 阶多项式,它的 \(n\) 次导函数的根也一定都是实数。Rolle 定理保证这些根全部落在 \(P(x)\) 的根包围的区间里。

    还要证明 \(P_n(x)\) 的根不是 \(1,-1\)。这个可以通过 \(n\) 次求导之后的形式确定。

  • 证明,\(L_n(x) = \dfrac{e^x}{n!} \dfrac{d^n}{dx^n}(e^{-x}x^n)\) 是多项式并且它所有的根都是正实数。

    因为这是 Laguerre 多项式!(暴论

    类似的考虑,也很 trivial.

  • 证明,\(H_n(x) = (-1)^n e^{x^2} \dfrac{d^n}{dx^n}(e^{-x^2})\) 是多项式并且它所有的根都是实数。

    因为这是 Hermite 多项式!(什么嘛,我的数值分析学得也没那么差嘛

    写一个递推:\(H_{n+1} (x)=2xH_n(x) - H_n' (x)\),所以是多项式。归纳地证明 \(H_n\) 的所有根都是实数时,\(H_{n+1}\) 的所有根都是实数。

Emile Borel 引理

第一部分:截断函数的构造

  • 定义函数 \(\phi\colon \mathbb R\rightarrow \mathbb R\)

    \(\phi(x)=\begin{cases}e^{-\frac{1}{x^2}}, \ \ x>0;\\0, \ \ \ \ x\leqslant 0.\end{cases}\)

    证明,\(\phi\in C^\infty(\mathbb R)\)

    只要证明在 \(x=0\) 处无限可微。实际上一阶导数是 \(0\),二阶及以上可以归纳地证明都是 \(0\)

  • 定义函数 \(\chi\colon \mathbb R\rightarrow \mathbb R\)\(\chi(x)=\frac{\phi(2-|x|)}{\phi(2-|x|)+\phi(|x|-1)}\),证明,\(\chi(x)\in C^\infty(\mathbb R)\) 并且 \(\chi\big|_{[-1,1]}\equiv 1\)\(\chi\big|_{(-\infty,-2]\cup [2,\infty)}\equiv 0\)\(0\leqslant \chi(x)\leqslant 1\) 并且是偶函数。

    \(\chi\big|_{[-1,1]}\equiv 1\)\(\chi\big|_{(-\infty,-2]\cup [2,\infty)}\equiv 0\) 这两个结论是显然的,所以在这几个区间上都是无限维可导。在 \([1,2]\) 上显然也无限维可导,所以只要证明在 \(1\)\(2\) 两点处无限维可导。(因为是偶函数,所以很容易可以看出负数区间上是一样的)。这两个也很好证明,一阶导数都是 \(0\)

  • 证明,对任意的 \(0<a<b\),存在光滑偶函数 \(\rho(x)\in C^\infty(\mathbb R)\),使得 \(\rho\big|_{[-a,a]}\equiv 1\)\(\rho\big|_{(-\infty,-b]\cup [b,\infty)}\equiv 0\)\(0\leqslant \rho(x)\leqslant 1\)

    构造 \(\rho (x) = \frac{\phi(b-|x|)}{\phi(b-|x|)+\phi(|x|-a)}\)

  • 证明,存在偶函数 \(\psi \in C^\infty(\mathbb R^n)\),使得 \(\psi\big|_{\{x\mid|x|\leqslant 1\}}\equiv 1\)\(\psi\big|_{\{x\mid|x|\geqslant 2\}}\equiv 0\)\(0\leqslant \psi(x )\leqslant 1\)

    构造 \(\psi (x) = \chi(|x|)\)

第二部分:逐项求导定理

\(I=[a,b]\) 是闭区间,\(\{f_k\}_{k\geqslant 0}\)\(C^1(I)\) 的一列函数,我们假设 \(\displaystyle \sum_{k=0}^\infty f_k\)\(I\) 上逐点收敛,即对任意的 \(x\in I\)\(\displaystyle \sum_{k=0}^\infty f_k(x)\) 收敛,我们记 \(f(x)=\displaystyle \sum_{k=0}^\infty f_k(x)\)

  • 我们假设函数级数 \(\displaystyle \sum_{k=0}^\infty f'_k(x)\)\(I\) 上绝对收敛,即数项的级数 \(\displaystyle \sum_{k=0}^\infty \|f'_k\|_\infty\) 收敛,其中 \(\displaystyle \|f\|_\infty=\sup_{x\in I}|f(x)|\)。证明,\(f\) 是可导的并且 \(f'(x)=\displaystyle \sum_{k=0}^\infty f'_k(x)\)。(提示:设法将求和拆 \(\displaystyle \sum_{k=0}^\infty = \sum_{k=0}^N+\displaystyle \sum_{k=N+1}^\infty\)

    \(\displaystyle \sum_{k=0}^\infty f'_k(x)\)\(I\) 上绝对收敛,所以 \(\displaystyle \sum_{k=0}^\infty f'_k(x)\)\(I\) 上逐点都是收敛的,上式右端一定收敛。所以只要证明这个等号成立。

    对任意的 \(\varepsilon >0\),找一个合适的 \(|x-x_0| < \delta\) 和一个合适的 \(N \in \mathbb Z+\),满足:

    \(|\frac{f_k(x)-f_k(x_0)}{x-x_0} - f_k '(x_0) | <\varepsilon\)\(k=0,1,2,...,N\) 都成立(导数的定义),而且 \(|\displaystyle \sum_{k=N+1}^\infty \|f'_k\|_\infty|<\varepsilon\)(收敛列的 Cauchy 性质)。

    于是 $|{k=0}^- {k=0}f'k(x)| |{k=0}N - {k=0}^N f'k(x)|+|{k=N+1}^- {k=N+1}^f'_k(x)| $。

    右边第一项的上界是 \(N\varepsilon\),第二项用一下 Lagrange 中值定理处理前一个级数和,可以知道上界是 \(2\varepsilon\)

    因此 \(\lim _{x \to x_0} \displaystyle \sum_{k=0}^\infty \frac{f_k(x)-f_k(x_0)}{x-x_0}- \displaystyle \sum_{k=0}^\infty f'_k(x) =0\),也就是 \(f'(x)=\displaystyle \sum_{k=0}^\infty f'_k(x)\),得证。

  • (逐项求导定理)\(I=[a,b]\) 是闭区间,\(\{f_k\}_{k\geqslant 0}\)\(C^1(I)\) 的一列函数,我们假设 \(\displaystyle \sum_{k=0}^\infty f_k\)\(I\) 上逐点收敛。如果函数级数 \(\displaystyle \sum_{k=0}^\infty f'_k(x)\)\(I\) 上一致收敛,那么 \(f\) 是可导的并且 \(f'(x)=\displaystyle \sum_{k=0}^\infty f'_k(x)\)

    证明 \(f\) 是可导的只需要证明导函数的这个式子成立。

    \(\displaystyle \sum_{k=0}^\infty f'_k(x)\)\(I\) 上一致收敛也可以推出,对 \(\varepsilon >0\),存在某个合适的 \(N\) 使得 \(|\displaystyle \sum_{k=N+1}^\infty f'_k(x)| < \varepsilon\) 对任意的 \(x\in I\) 都成立,所以前一题不等式里的第二项仍然被 \(2\varepsilon\) 控制,不等式仍然可以成立。

    同时,\(f'(x)=\displaystyle \sum_{k=0}^\infty f'_k(x)\) 右端也对任意的 \(x \in I\) 都是收敛级数,是良定义的。

  • 试利用逐项求导定理计算 \(e^x\) 的导数。

    Trivial,无穷级数求导之后形式仍然相同。

第三部分:Borel引理的证明

我们现在任意给定一个数列 \(\{a_k\}_{k\geqslant 0}\)

  • 对任意给定的正数 \(\lambda_k>0\),试计算函数 \(f_k(x)=\dfrac{a_k}{k!}x^k\chi(t_k x)\)\(x=0\) 处的任意阶导数(包括零阶)。

    \(n\) 阶导数:$n k $ 时是 \(0\)\(n=k\) 时是 \(a_k\)

  • 证明,当 \(k\geqslant 2n\) 时,我们有 \(f_k^{(n)}(x)=a_k\sum_{\ell=0}^n{n \choose \ell}\frac{t_k^{n-\ell}}{(k-\ell)!}x^{k-\ell}\chi^{(n-\ell)}(t_k x)\)

    归纳就好了。

  • (Borel引理)任意给定一个数列 \(\{a_k\}_{k\geqslant 0}\),证明,存在 \(\mathbb R\) 上的光滑函数 \(f\),使得对任意的 \(k\geqslant 0\)\(f^{(k)}(0)=a_k\)。(提示:略难。)

    \(f(x) = \displaystyle \sum_{k=0}^\infty f_k(x)\),如果它可以逐项求导,那么是能够满足后一个条件的,所以只要证明 \(f(x)\)\(0\) 处的导数可以通过级数的逐项求导得到。另外想证明它是光滑函数的话,要归纳证明每一阶导函数都可以继续逐项求导。

    注意这里还有个未知的数列 \(t_k\) 可以用来取值并控制这一函数,所以只要证明对任意的 \(n \in \mathbb Z +\),有 \(\displaystyle \sum_{k=0}^\infty f^{(n)}_k(x)\) 是在 \(I=[-a,a]\) 上一致收敛的。

    \(k\geq 2n\) 时,\(\| f^{(n)}_k \| _\infty \leq a_k A \sum_{\ell=0}^n{n \choose \ell}\frac{t_k^{n-\ell}}{(k-\ell)!}x^{k-\ell} \leq a_kA \sum_{\ell=0}^n{n \choose \ell}t_k^{n-\ell}a^{k-\ell}= a_kAa^k (\frac 1 a + t_k)^n\)。其中我们取 \(a = \frac{1}{t_k}\),于是右边可以继续放缩成 \(A2^n t_k^n (a_k \frac{1}{t_k ^k})\),只要取 \(t_k = 2|a_k| +1\) 就能保证 \(\displaystyle \sum_{k=2n}^\infty \| a_k \frac{1}{t_k ^k} \|\) 是收敛的。

    于是 $_{k=0}^| f{(n)}k(x) | = {k=0}{2n-1} | f{(n)}k(x) | + {k=2n}| f^{(n)}_k(x) | $ 右边两项一个有限,一个一致收敛,因此总体也是一致收敛的。于是有 \(\displaystyle \sum_{k=0}^\infty f^{(n)}_k(x)\) 是在 \(I=[-a,a]\) 上一致收敛的。

第四部分:Peano对Borel引理的证明(选做部分,不交作业)

  • \(\{c_k\}_{k\geqslant 0}\)\(\{b_k\}_{k\geqslant 0}\) 是两个序列,其中 \(b_k\) 都是正数。证明,

    \(\Bigl(\frac{c_k x^k}{1+b_k x^2}\Bigr)^{(n)}(0)=\begin{cases}n!(-1)^jc_{n-2j}b_{n-2j}^j, \ \text{若}~k=n-2j, j\in \mathbb{Z}_{\geqslant 0};\\0, \ \ \ \text{其它情形}.\end{cases}\)

    Trivial.

  • 证明,存在常数 \(C\),对任意的 \(k\geqslant n+2\),对任意的 \(x\),我们有

    \(\Big|\Bigl(\frac{c_k x^k}{1+b_k x^2}\Bigr)^{(n)}(x)\Big|\leqslant C(n+1)!\frac{|c_k|k!}{b_k}|x|^{k-n-2}\)

    李在赣神魔.jpg

  • 证明,当 \(\{c_k\}_{k\geqslant 0}\) 给定的时候,我们可以选取 \(b_k\),使得 \(b_k\) 仅依赖于 \(c_k\) 的选取,并且函数级数 \(\displaystyle f(x)=\sum_{k=0}^\infty \frac{c_k x^k}{1+b_k x^2}\) 是无限次可微分的。

    如果我们只要求 \(I=(-1,1)\) 的区间应该是可以做到的,让 \(b_k = \frac{|c_k| k!}{(n+1)!}\) 就可以保证 \(\displaystyle \sum_{k=n+2}^\infty \frac{c_k x^k}{1+b_k x^2}\)\(n\) 阶微分是收敛列,对于任意取的 \(n \in \mathbb Z+\) 可以知道 \(\displaystyle \sum_{k=0}^{n+1} \frac{c_k x^k}{1+b_k x^2}\)\(n\) 阶微分是有限的。所以 \(f^{(n)}(x)\) 存在。

  • 证明,\(f(0)=c_0\)\(f'(0)=c_1\),并且当 \(n\geqslant 2\) 时,我们有

    \(\frac{f^{(n)}(0)}{n!}=d_n+\sum_{j=1}^{[\frac{n}{2}]}(-1)^jc_{n-2j}b_{n-2j}^j\)

    第一问给出了,trivial.

    这里这个 \(d_n\) 或许就是 \(c_n\)

  • 证明,我们可以通过恰当的选取 \(\{c_k\}_{k\geqslant 0}\)\(\{b_k\}_{k\geqslant 0}\) 来证明Borel引理。

    李在赣神魔.jpg

    \(f\) 是光滑函数已经证明了,在第三问里面我们要求 \(b_k\) 是依赖于 \(c_k\) 的数列,因此只要求出所有的 \(c_k\) 就可以了。

    对简单的情形,取 \(c_0 = a_0\)\(c_1 = a_1\)\(n=2\) 时可以推出 \(a_2 = 2(c_2 - c_0 b_0)\),其中 \(c_0,b_0\) 都已知,可以推出 \(c_2\)。找规律就知道每个 \(\frac{f^{(n)}(0)}{n!}=d_n+\sum_{j=1}^{[\frac{n}{2}]}(-1)^jc_{n-2j}b_{n-2j}^j\) 式中都可以推出新的 \(c_n\),依靠的是已知的 \(c_0,c_1,...,c_n\) 的信息。此时 \(f\) 满足 Borel 引理,得证。

梦话 3.0:为什么积分章节的习题看起来还是导数

如果不另加说明,\(f\) 总代表某区间 \(I\) 上实值函数。

如果对任意的 \(x,y \in I\)\(t \in (0,1)\),都有 \(f(tx + (1-t)y) < tf(x) + (1-t)f(y)\),我们就称 \(f\) 为严格凸的。仿照课堂上凹函数的定义,我们可以类似地定义严格凹的函数(即 \(-f\) 是严格凸的)。

凸函数的基本性质

  • 试证明凸函数的如下基本性质:

    • 如果 \(f,g\) 是区间 \(I\) 上的凸函数,那么 \(f+g\) 也是凸函数。

      Trivial.

    • 如果 \(f,g\) 是 区间 \(I\) 上的单调递增的非负的凸函数, 那么 \(fg\) 是凸函数。

      Trivial.

    • 如果 \(f\) 是 区间 \(I\) 上的凸函数,\(g\) 是 区间 \(J \supset f(I)\) 上的单调递增凸函数, 那么 \(g \circ f\) 是凸函数。

      Trivial.

    • 如果 \(f,g\) 是 区间 \(I\) 上的凸函数, 那么 \(h(x) = \max\{f(x),g(x)\}\) 是凸函数。

      Trivial.

  • 假设 \(f\in C\bigl((a,b)\bigr)\)。如果对任意的 \(x,y \in (a,b)\),都有 \(\displaystyle f\left(\frac{x+y}{2}\right) \leqslant \frac{f(x) + f(y)}{2}\),证明,\(f\) 是凸函数。

    对任意的有理数可以用二进制,无理数点可以用有理数的稠密性逼近。

  • \(f\)\([a,b]\) 上的凸函数。如果存在 \(c \in (a,b)\),使得 \(f(c) \geqslant \max\{f(a),f(b)\}\)。试证明,\(f\) 是常值函数。

    Trivial.

  • \(f\)\(\mathbb{R}\) 上的凸函数。如果 \(f\)\(\mathbb{R}\) 上有界,证明,\(f\) 是常值函数。

    如果不是常值函数,也就是存在 \(f(a) = f(b)\),那对于第三个点 \(c \to \infty\) 一起比较斜率就知道 \(f(c)\) 无界了。

  • \(f\) 是区间 \(I\) 上的严格凸函数。假设 \(f(x_0) \in I\)\(f\) 的局部极小值,证明,\(x_0\)\(f\) 唯一的整体极小值点,即对任意的 \(x \in I-\{x_0\}\),我们有 \(f(x_0) < f(x)\)

    局部极小值的意思是存在一个邻域 \(J \subset I\) 使得 \(f(x_0)\) 在其中最小。取 \(x_1 \in (x_0,x_0 +\delta)\)\(x_2 \notin J\)\(f(x_2) \leq f(x_0)\),比较三者的斜率知道不符合严格凸条件。

  • \(I\) 是开区间。证明,\(f\) 是凸函数等价于对任意点 \(x_0 \in I\),存在实数 \(a \in \mathbb{R}\),使得对任意的 \(x \in I\),我们都有 \(f(x) \geqslant a(x - x_0) + f(x_0)\)

    正推取 \(a = f'(x_0)\),反推考虑 \(x_0\) 和左右两点组成的三点的斜率关系。

凸函数与不等式

  • 试求所有的正数 \(a\),使得不等式 \(a^x \geqslant x^a\) 对任意的 \(x > 0\) 都成立。

    \(x=a\) 是函数 \(f(x) = a^x - x^a\) 的一个零点,于是它也是一个极小值点,此处导数值为 \(0\) 且二阶导数为正。

    于是 \(a=e\)

  • 证明如下不等式并给出等号成立的条件:对任意正数 $x_i $ 和 $t_i \((\)i=1,,n\(),\)_{i=1}^n t_i = 1$,都有

    $( {i=1}^n t_i x_i )^{ {i=1}^n t_i x_i} _{i=1}^n x_i^{t_i x_i} $

    两边取对数然后用 Jensen 不等式。

  • 证明Young不等式并给出等号成立的条件:对于任意正数 \(a,b\),任意的实数 \(p,q\),其中 \(\dfrac{1}{p} + \dfrac{1}{q} = 1\)(我们要求 \(p\)\(q\) 不是 \(0\) 或者 \(1\)),我们有

    \(ab \leqslant \frac{a^p}{p} + \frac{b^q}{q}, \ \ \text{如果}~p > 1; \ \ ab \geqslant \frac{a^p}{p} + \frac{b^q}{q}, \ \ \text{如果}~p < 1\)

    这个高中竞赛的时候就证明过,咕了。

  • 证明 Holder 不等式并给出等号成立的条件:设 \(x_i,y_i \geqslant 0\),其中 \(i = 1,\cdots,n\)\(p,q \neq 0,1\),其中 $ + = 1$,我们有

    \(\sum_{i=1}^{n} x_i y_i \leqslant \left(\sum_{i=1}^{n}x_i^p\right)^{1/p}\left(\sum_{i=1}^{n}y_i^q\right)^{1/q}, \ \text{如果}~p > 1; ~\sum_{i=1}^{n} x_i y_i \geqslant \left(\sum_{i=1}^{n}x_i^p\right)^{1/p}\left(\sum_{i=1}^{n}y_i^q\right)^{1/q},\ \text{如果}~p < 1\)

    在上述不等式中,如果 \(p < 0\),我们假设 \(x_i > 0\)。(这个不等式的结论比证明重要得多,同学们可以参考其它资料来写下证明)

    高中竞赛的时候也证明过,咕了。

利用(高阶)导数的信息刻画函数(低阶导数)

在本习题中,我们把函数 \(f\) 也记为 \(f^{(0)}\)

  • 我们假设 \(f\in C\bigl([0,1]\bigr)\)\(g\)\([0,1]\) 上可导且 \(g(0) = 0\)。如果存在常数 \(\lambda \neq 0\),使得对任意的 \(x\in [0,1]\),都有 \(|g(x)f(x) + \lambda g'(x)| \leqslant |g(x)|\), 证明,\(g(x) \equiv 0\)

  • \(f\)\((-1,1)\) 上二阶可导,\(f(0) = f'(0) = 0\)。如果对任意的 \(x\in (-1,1)\),都有 \(|f''(x)| \leqslant |f(x)|+|f'(x)|\),证明,\(f(x) \equiv 0\)

  • \(f\)\(\mathbb R\) 上二阶可导,满足 \(f(0) = f'(0) = 0\), 且存在正实数 \(C\) 使得, 对所有的 \(x\in \mathbb R\), 有 \(|f''(x)| \le C|f(x)f'(x)|\)。证明,\(f(x) \equiv 0\), \(\forall x \in \mathbb R\)

  • \(n\) 是正整数,\(f\)\(\mathbb R\)\(n\) 阶可导,\(f(0) = f'(0) = \cdots = f^{(n-1)}(0) = 0\)。如果 \(C\in\mathbb{R}_{>0}\)\(\ell \in \mathbb{Z}_{\geqslant 0}\),使得对任意 \(x\in \mathbb R\) 都有 \(|f^{(n)}(x)| \leqslant C|f^{(\ell)}(x)|\),证明,\(f(x) \equiv 0\)

  • \(n\) 是正整数,证明,多项式 \(\displaystyle P(x) = \sum_{k=0}^{n+1} C_{n+1}^k (-1)^k (x-k)^n\)\(0\)

    考虑 \(x^i\) 的系数,全都是 \(0\)。(好怪

  • \(f\in C^\infty(\mathbb R)\)。假设存在正实数 \(C\),使得对任意 \(n \in \mathbb{Z}_{\geqslant 0}\) 和任意的 \(x \in \mathbb R\),我们都有 \(|f^{(n)}(x)| \leqslant C\)

    • 证明,给定任意的 \(x_0 \in \mathbb R\),我们可以(以 \(x_0\) 为中心)将 \(f(x)\)\(\mathbb R\) 上展开为无穷Taylor级数,即\(f(x) = \sum_{k=0}^\infty \frac{f^{(k)}(x_0)}{k!}(x-x_0)^k, \qquad \forall x \in \mathbb R\)

      只要证明这个级数是逐点收敛的,实际上 \(|f(x) |\leq C \exp(x-x_0)\)

    • \(E\subset \mathbb R\) 是一个无穷集并且有界。证明,如果 \(f\)\(E\) 上的取值都是零,那么 \(f\equiv 0\)

  • 假设 \(f\in C^2\bigl((0,1)\bigr)\)\(\displaystyle\lim_{x\to 1^-}f(x) = 0\)。如果存在 \(C>0\),使得对任意 \(x\in (0,1)\),我们都有不等式 \((1-x)^2|f''(x)| \leqslant C\)。证明,\(\displaystyle\lim_{x\to 1^-} (1-x)f'(x) = 0\).

函数的重根

上传者注:本题题设条件下可以举出很多反例,建议读者将可微条件全部改为足够高阶连续可微,或者光滑。

如果 \(f\)\(x_0\) 的一个邻域内满足 \(f(x) = (x-x_0)^r g(x)\),其中 \(r \in \mathbb{Z}_{\geqslant 0}\)\(g\)\(x_0\) 处连续并且 \(g(x_0) \neq 0\),那么我们就称 \(x_0\)\(f\)\(r\)-重根。我们注意到,\(0\)-重根并非根。

  • 假设 \(x_0\)\(f\)\(r\)-重根,其中 \(r\geqslant 1\)。证明,如果 \(g(x) = \dfrac{f(x)}{(x-x_0)^r}\) 可微,那么 \(x_0\)\(f'\)\((r-1)\)-重根。

    \(f'(x) = (x-x_0)^r g'(x) - rg(x) (x-x_0)^{r-1}\)

  • 假设 \(f\)\(\mathbb R\)\(n\) 阶的可微函数。证明,如果 \(f(x) = 0\)\(n+1\) 个不同的实根,那么 \(f^{(n)}(x) = 0\) 至少有一个实根。

    不停用 Lagrange 中值定理。

  • \(f\)\(\mathbb R\) 上的可微函数。假设 \(f(x) = 0\) 按重数计算恰有 \(r\) 个实根,也就是说,\(f(x) = 0\)\(s\) 个相异的实根 \(x_1, x_2, \cdots, x_s\),它们的重数分别为 \(r_1, r_2, \cdots, r_s\) 并且 \(r_1+r_2 + \cdots + r_s = r\)。证明,\(f'(x) = 0\) 按重数计至少有 \(r-1\) 个根。

    \(f'(x)\) 对于相异的实根 \(x_1, x_2, \cdots, x_s\),它们的重数分别为 \(r_1-1, r_2-1, \cdots, r_s-1\) ,并且在 \(x_1, x_2, \cdots, x_s\) 之间用 Lagrange 中值定理还有 \(s-1\) 个根,于是共有 \(r-1\) 个根。

  • 假设 \(f\)\(\mathbb R\)\(n\) 阶的可微函数。证明,如果 \(f(x) = 0\) 按重数计恰有 \(n+1\) 个实根,那么 \(f^{(n)}(x) = 0\) 至少有一个实根。

    归纳。

积分

2023 年 2 月 8 日,星期三,学了半天积分结果配套的作业题还是导数题,摸不着头脑.jpg

Riemann 积分

  • 阶梯函数是最简单的一类积分函数,它的积分值对于任意相容的分划都是相等的。这是一个很美妙的证明,用到了一个简单的逻辑:$$ 和 \(\sigma '\) 的并 \(\sigma \cup \sigma '\) 是二者的一个加细;如果 \(\sigma_1\)\(\sigma _2\) 的一个加细且它们都是相容的分割,那么二者的积分值相等;因此 \(\sigma\)\(\sigma '\) 的积分都和 \(\sigma \cup \sigma '\) 的积分相等,由任意性可知任意划分的积分都相等。

    阶梯函数的积分是满足线性性的,它还满足一些看起来很显然但其实证明也不是很难的性质,略了。

  • 为了扩大可以“积分”的函数的种类(目前只有阶梯函数积分),试一下用阶梯函数积分逼近一般的连续函数。

    一个技术性的引理(定义),实际上类似于极限的 \(\delta - \varepsilon\) 描述和点列描述法:

    \(I=[a,b]\) 是有界闭区间,\(f\colon I\rightarrow \mathbb R\) 是函数,如下命题是等价的:

    • 对任意的 \(\varepsilon>0\),存在两个阶梯函数 \(F_\varepsilon\colon I \rightarrow \mathbb R\)\(\Psi_\varepsilon\colon I \rightarrow \mathbb R\),使得对任意的 \(x\in I\),都有

      \(\big|f(x)-F_\varepsilon(x)\big|<\Psi_\varepsilon(x)\) 并且 \(\int_I \Psi_\varepsilon <\varepsilon\)

    • 存在两个阶梯函数的序列 \(\{f_n\}_{n\geqslant 1} \subset \mathcal{E}(I)\)\(\{\psi_n\}_{n\geqslant 1} \subset \mathcal{E}(I)\),使得对任意的 \(x\in I\),都有

      \(\big|f(x)-f_n(x)\big|<\psi_n(x)\) 并且 $ _{n}_I _n =0$

    满足二者之一时,称 \(f\) 可以被阶梯函数逼近,是区间 \(I\) 上 Riemann 可积的函数。\(\mathcal R (I)\) 表示全体 \(I\) 上 Riemann 可积的函数。此时可以定义 \(f\) 的 Riemann 积分:\(\int_{I}=\int_{a}^b\colon \mathcal{R}(I)\rightarrow V, \ \ f\mapsto \lim_{n\rightarrow \infty} \int_a^b f_n\)

    关于这个定义为什么是良定义的问题,首先要证明上述极限存在,再证明 \(f\) 的积分定义不依赖于具体的 \(\lbrace f_n \rbrace\)\(\lbrace \psi_n \rbrace\),实际上它们都是简单的阶梯函数,所以由定义都是容易证明的。

  • 研究一下 \(I\) 上的 Riemann 可积空间 \(\mathcal R (I)\) 的性质,由于我之前根本没学会这个,还是写细一点。

    比较容易证明的几个性质是:\(\mathcal R (I)\) 是实数域上的线性空间,\(\mathcal R(I)\) 上的函数都有界(注意 \(I\) 还是一个闭区间的情况,这里还没有研究到广义积分),阶梯函数都在 \(\mathcal R (I)\) 中,Riemann 可积函数取绝对值也还是 Riemann 可积的,乘积也是 Riemann 可积的。

    其他的性质有:\(\mathcal R (I)\) 包含所有的 \(C(I)\) 上函数(利用一致连续性手动切出逼近的阶梯函数),有限维的情况。

    \(C(I)\subset \mathcal{R}(I)\)

    假设 \(f\in C(I)\),根据 \(f\) 的一致连续性,对任意的 \(\varepsilon>0\),存在 \(n\in\mathbb{Z}_{\geqslant 1}\),使得对任意的 \(x,y\in I\),当 \(|x-y|<\dfrac{1}{n}\) 时,我们有 \(|f(x)-f(y)|<\frac{\varepsilon}{b-a}\)。此时,我们令

    \(F(x)=\sum_{k=1}^n f\bigl(a+k\frac{b-a}{n}\bigr)\mathbf{1}_{[a+(k-1)\frac{b-a}{n},a+k\frac{b-a}{n}]}(x)\)

    其中,\(\mathbf{1}_{[a+(k-1)\frac{b-a}{n},a+k\frac{b-a}{n}]}\) 是示性函数。这个 \(F(x)\) 显然是阶梯函数。由一致连续性,我们知道

    \(\big|f(x)-F(x)\big|<\Psi_\varepsilon(x)\equiv \frac{\varepsilon}{b-a}\),所以,\(\int_a^b \Psi_\varepsilon(x) =\varepsilon\)。从而,\(f\) 是Riemann可积的函数。

Darboux 上下和

这是一个刻画 Riemann 可积的实数值函数的方法。

当然,刻画 Riemann 可积函数的方式一共有四种,彼此等价,具体就不说了,可见:Read More

梦话:终于找到积分的作业在哪了但是还没写

积分定义的补充与扩展

我们总假设 \(I=[a,b]\subset \mathbb R\) 是一个有界闭区间,\(V\) 是一个赋范线性空间。

  • \(\sigma_1,\sigma_2\in \mathcal{S}\) 是两个分划。证明,对任意的 \(\varepsilon>0\),总存在分划 \(\sigma\),使得 \(\sigma\prec \sigma_1\)\(\sigma\prec \sigma_2\) 并且它的步长 \(|\sigma|<\varepsilon\)

    \(\sigma_1 \cup \sigma_2\) 再加细就可以了。

  • 考虑在 \(V\) 中取值的阶梯/简单函数的空间 \(\mathcal{E}(I)\),证明,这是 \(\mathbb R\)-线性空间并且积分算子 \(\displaystyle\int_{a}^b\colon \mathcal{E}(I)\rightarrow V\) 是良好定义的(不依赖于分划的选取)并且是线性映射,其中积分的定义方式与函数在 \(\mathbb R\) 中取值时的方式一致。据此,用阶梯函数逼近的方式定义在 \(V\) 中取值的Riemann可积的函数。你不需要写下细节但是你应该对照笔记研究原来证明的每一步。

  • 假设 \(f\colon I\rightarrow \mathbb{R}^n\),其中 \(f_i\)\(f\) 的每个分量。那么 \(f\in \mathcal{R}(I)\) 当且仅当对每个分量 \(f_i\) 我们都有 \(f_i\in \mathcal{R}(I)\)。特别地,当 \(f\colon \mathbb R\rightarrow \mathbb{C}\) 时,我们有 \(\displaystyle \int_a^b f = \int_a^b \mathbb Re f+i\int_{a}^b \Im f\),其中 \(\mathbb Re f\)\(\Im f\) 分别为 \(f\) 的实部和虚部。

  • 试证明积分的区间可加性:假设 \(a<c<b\),那么对于任意的 \(f\in \mathcal{R}(I)\),我们有 \(f\)\([a,c]\)\([c,b]\) 上的限制都是阶梯函数,并且 \(\int_{a}^b f =\int_{a}^c f+\int_c^b f\)

  • 证明,对于任意两个分划 \(\sigma\)\(\sigma'\),它们所对应的Darboux上下和满足

    \(\underline{S}(f;\sigma)\leqslant \overline{S}(f;\sigma')\)

    据此证明,如果 \(f\in \mathcal{R}(I)\),就有 \(\displaystyle\lim_{|\sigma|\rightarrow 0}|\underline{S}(f;\sigma)-\overline{S}(f;\sigma)|=0\),即对任意的 \(\varepsilon>0\),一定存在 \(\delta>0\),对任意的分划 \(\sigma\),只要 \(|\sigma|<\delta\),我们就有 \(|\underline{S}(f;\sigma)-\overline{S}(f;\sigma)|<\varepsilon\)

  • \(f\in \mathcal{R}(I)\)。证明,改变 \(f\) 在有限个点上的取值所得到的函数仍是Riemann可积的并且积分与 \(f\) 的相同。

  • \(f\in C([a,b])\)。假设对任意的 \(x\in I\),我们都有 \(f(x)\geqslant 0\) 并且存在点 \(x_0 \in I\) 使得 \(f(x_0) > 0\)。 证明,\(\displaystyle\int_a^b f > 0\)

  • (不定积分的分部积分公式:对计算不定积分有用)假设 \(f,g\in C^1(I)\),那么,我们有

    \(\int f'\cdot g=f\cdot g-\int f\cdot g'\)

  • (不定积分的变量替换公式:对计算不定积分有用)假设 \(\Phi\colon \mathbb R\rightarrow \mathbb R\) 可微,\(f\) 是连续函数,那么

    \(\int (f\circ \Phi)\Phi'=\int f\)

思考题:多项式逼近和Weierstrass--Stone定理

我们要证明如下著名的定理:任意给定有界闭区间 \(I=[a,b]\) 上的连续函数,我们总是可以用一个多项式来足够好地逼近它。更精确地说,给定 \(f\in C([a,b])\),对任意的 \(\varepsilon>0\),存在多项式 \(P_\varepsilon\),使得 \(\displaystyle \sup_{x\in [a,b]}|f(x)-P_\varepsilon(x)|<\varepsilon\),即若用 \(P([a,b])\) 表示 \([a,b]\) 上多项式函数组成的空间,则 \(P([a,b])\) 在度量空间 \(C([a,b])\) 中稠密(\(C([a,b])\) 上用的范数是 \(\|f\|_\infty=\displaystyle \sup_{x\in [a,b]}|f(x)|\))。

Dini定理及应用

  • (Dini定理)假设 \(K\subset \mathbb R^n\) 是紧子集,\(f_n\colon K\rightarrow \mathbb R\) 是一列连续函数,它们逐点地收敛到连续函数 \(f\colon K\rightarrow \mathbb R\),即对每个 \(x\in K\),我们都有 \(\displaystyle \lim_{n\rightarrow \infty}f_n(x)=f(x)\)。证明,如果 \(\{f_n\}_{n\geqslant 1}\) 是上升的函数列(即对任意 \(x\in K\)\(n\),我们都有 \(f_n(x)\leqslant f_{n+1}(x)\)),那么 \(f_n\) 一致收敛到 \(f\)。(参考荆公子的某次习题课)

  • 考虑区间 \(I=[-1,1]\)。我们通过归纳的方式定义一族多项式函数:

    \(P_0(x)=0, \ \ P_{n+1}(x)=P_n(x)+\frac{1}{2}(x^2-P_n^2(x))\)

    证明,对任意的 \(n\)\(x\),我们都有 \(0\leqslant P_n(x)\leqslant P_{n+1}(x)\leqslant |x|\)

  • 证明,绝对值函数 \(|x|\)\(I=[-1,1]\) 上可以被多项式一致地逼近,即对任意的 \(\varepsilon>0\),存在某个多项式函数 \(P_\varepsilon(x)\),使得 \(\displaystyle \sup_{x\in [-1,1]}\big||x|-P_\varepsilon(x)\big|<\varepsilon\)

区间上的情形

这一部分中,我们假设 \(I=[0,1]\)\(n\) 是正整数。

  • 对任意的 \(0\leqslant k\leqslant n\),我们定义 \(p_{n,k}(x)=\displaystyle{n \choose k}x^k(1-x)^{n-k}\)。证明,\(\displaystyle \sum_{0\leqslant k \leqslant n}p_{n,k}(x-\frac{k}{n})^2=\frac{x(1-x)}{n}\)

  • 任意给定 \(f\in C([0,1])\),我们定义 \(B_{f,n}(x)= \displaystyle\sum_{0\leqslant k \leqslant n}f\bigl(\frac{k}{n}\bigr){n \choose k}x^k(1-x)^{n-k}\)。对 \(x\in [0,1]\),证明,

    \(|f(x)-B_{f,n}(x)|\leqslant \sum_{k=0}^n \bigl|f(x)-f\bigl(\frac{k}{n}\bigr)\bigr|p_{n,k}(x)\)

  • (用Bernstein多项式逼近连续函数)任意给定 \(f\in C([0,1])\),证明,对任意 \(\varepsilon>0\),总存在 \(n\),使得 \(\|f-B_{f,n}\|_\infty <\varepsilon\)

Remark:用概率论的观点,\(x\in [0,1]\) 给定,Bernstein多项式逼近的的方法讲的是概率测度

\(\mu_x=\displaystyle\sum_{k=0}^n {n \choose j}x^k(1-x)^{n-k}\delta_{\frac{k}{n}}\) 的极限是Dirac测度 \(\delta_x\)

紧集上的情形

从此往后,我们假设 \(K\subset \mathbb R^n\) 是紧集,\(C(K)\)\(K\) 上的实数值连续函数所构成的线性空间,用 \(P(K)\) 表示 \(K\) 上多项式函数组成的空间(即形如 \(\displaystyle \sum_{|\alpha|\leqslant m} c_\alpha x^\alpha\) 的函数,其中 \(m\) 是正整数,\(\alpha\) 是多重指标,请参见关于多重偏导数的课堂笔记)。

  • 假设 \(\mathcal{A}\subset C(K)\) 是非零的线性子空间。如果对任意的 \(f,g\in \mathcal{A}\),它们的乘积 \(f\cdot g \in \mathcal{A}\),我们就把 \(\mathcal{A}\) 称作是 \(C(K)\) 的一个子代数。证明,\(P(K)\)\(C(K)\) 的子代数。

  • 非零的线性子空间 \(\mathcal{A}\subset C(K)\) 是闭子代数,也就是说如果 \(\{f_k\}_{k \geqslant 1}\subset \mathcal{A}\)\(f_k\) 一致收敛到 \(f\),那么 \(f\in \mathcal{A}\)。假设常值函数 \(1\in \mathcal{A}\),证明,如果 \(f\in \mathcal{A}\),那么 \(|f|\in \mathcal{A}\)。(提示: 利用 W3)

  • 假设 \(\mathcal{A}\subset C(K)\) 是子集,如果对任意的 \(x,x'\in K\)\(x\neq x'\),都存在 \(f\in \mathcal{A}\),使得 \(f(x)\neq f(x')\),我们就称 \(\mathcal{A}\) 是能够区分点的。证明,\(P(K)\) 是能够区分点的。

  • 假设 \(f,g\in C(K)\)。证明,函数 \(f\wedge g (x)=\min\{f(x),g(x)\}\)\(f\vee g (x)=\max\{f(x),g(x)\}\) 都是连续的。

  • 假设 \(\mathcal{A}\subset C(K)\) 是子集,如果对任意的 \(f,g\in \mathcal{A}\)\(f\wedge g, f\vee g \in \mathcal{A}\),我们就称 \(\mathcal{A}\)\(\wedge\vee\)-封闭的。证明,\(\overline{P(K)}\)\(\wedge\vee\)-封闭的,其中 \(\overline{P(K)}=\bigl\{f\in C(K)\bigm|\text{存在}\{f_k\}_{k\geqslant1}\subset P(K), f_k\text{一致收敛到}f\bigr\}\)\(P(K)\)\(C(K)\) 中的闭包。

  • 我们现在假设 \(\mathcal{A}\subset C(K)\)\(\wedge\vee\)-封闭的,并且对任意的 \(\alpha,\beta \in \mathbb R\),任意的 \(x,y \in K\)\(x\neq y\)(我们此时假设 \(K\) 至少含有两个点)都存在函数 \(\varphi\in A\),使得 \(\varphi(x)=\alpha\)\(\varphi(y)=\beta\)

    我们通过下面的步骤来证明 \(\mathcal{A}\subset C(K)\) 是稠密的(即对任意给定的连续函数 \(f\in C(K)\),对任意的 \(\varepsilon>0\),总存在 \(\varphi \in \mathcal{A}\),使得 \(\|\varphi-f\|_{\infty}<\varepsilon\)):

    • 固定 \(x_0\in X\),对任意的 \(y\in K\),我们选取 \(\varphi_y\in \mathcal{A}\),使得 \(\varphi_y(x_0)=f(x_0)\) 并且 \(\varphi_y(y)=f(y)\),这样我们得到一族 \(\{f_y\in \mathcal{A}\mid y\in K\}\),据此,对每个 \(y\in K\),可以定义 \(U_y=\{z\in K\mid \varphi_y(z)>f(z)-\varepsilon\}\)。证明,存在有限个 \(y_1,\cdots,y_m\in K\),使得 \(K\subset U_{y_1}\cup \cdots \cup U_{y_m}\)
    • 证明,\(\varphi_{x_0}=\displaystyle\sup_{x\in K}\{\varphi_{y_1}(x),\cdots, \varphi_{y_m}(x)\}\in \mathcal{A}\) 并且对任意的 \(x\in K\),我们都有 \(\varphi_{x_0}(x)-f(x)>-\varepsilon\)
    • 证明,存在 \(\varphi \in \mathcal{A}\),使得对任意的 \(x\in K\),都有 \(|\varphi(x)-f(x)|<\varepsilon\)
  • 证明,如果子代数 \(\mathcal{A}\subset C(K)\) 是能够区分点的,\(\wedge\vee\)-封闭的并且包含所有的常数值函数,那么 \(\mathcal{A}\subset C(K)\) 是稠密的。

  • (Weierstrass--Stone定理)如果 \(\mathcal{A}\subset C(K)\) 是一个能区分点的子代数并且包含常值函数 \(1\),那么 \(\mathcal{A}\subset C(K)\) 是稠密的。

Remark:我们还有一种复值函数的Weierstrass--Stone定理,在很多的场合有着重要的应用,有兴趣的同学可以自己查阅。

  • 多项式函数 \(P(K)\) 在连续函数空间 \(C(K)\) 中是稠密的。

  • 给定以 \(2\pi\) 为周期的连续函数 \(f\in C(\mathbb R)\)。证明,任给 \(\varepsilon>0\),总存在一个有限的三角级数

    \(T(x)=\sum_{-N\leqslant k \leqslant N} a_k \cos(kx)+\sum_{-N\leqslant k \leqslant N} b_k \sin(kx)\)

    其中 \(N\) 是正整数,\(a_k,b_k\) 是实数,使得对任意的 \(x\in \mathbb R\),我们都有 \(|f(x)-T(x)|<\varepsilon\)。(提示:考虑 \(C([0,2\pi])\) 和它的某个子代数)

我很可爱 请给我钱(?)

欢迎关注我的其它发布渠道